Recent studies indicate a correlation between damage to human chromosome number six and adult schizophrenia. We know,...

Gerson on September 11, 2016

Question 11

why is the answer A and not C?

Replies
Create a free account to read and take part in forum discussions.

Already have an account? log in

Billy on May 26, 2017

Or why is it not E?

Mehran on May 31, 2017

@Goisonberry what would be the unrepresentative sample population here?

The samples are people without damage to chromosome number six and people with damage to this chromosome.

In terms of (E), the argument is actually NOT presuming that correlation implies causation.

Notice the recent studies indicate a correlation between damage and adult schizophrenia, but the conclusion here is "so there is no casual connection between damage to human chromosome number six and adult schizophrenia."

Hope that helps! Please let us know if you have any other questions.

Halle on April 2, 2018

I still don't understand your explanation for answer choice E.. Could you please elaborate?

Jameka on January 15, 2019

I selected answer choice D. incorrectly and I feel like I run into a few questions like this. Can you please elaborate on how to approach this question?
Thanks!

Ravi on January 15, 2019

@hallerae and @JCarter,

Happy to help! In the stimulus, we're told that there's a correlation
between damage to human chromosome number 6 and adult schizophrenia.

DHC6 - corr - AS

Keep in mind with correlations, there are four possible explanations:

1) DHC6 could be causing AS (A causing B)
2) AS could be causing DHC6 (B causing A)
3) X could be causing both DHC6 and AS (X causing A and B)
4) The correlation is purely coincidence

The stimulus then tells us that there are people without DHC6 who
develop AS and some people with DCH6 don't develop AS

/DHC6 and AS
DCH6 and /AS

The argument then concludes that there is no causal connection between
DHC6 and AS

not DHC6 - cause - AS

The argument is basically concluding that despite the correlation,
there is no causation going on. But what if there are different types
of damage to HC6? Couldn't that help explain why there's a correlation
between DHC6 and AS but then also some instances in which there is
DHC6 but /AS?

The question asks us to identify the flaw in the argument. It's hard
to see the flaw at first, so let's go to the answers to see what they
say. Even if the flaw is hard for us to find, we can still work toward
eliminating the four wrong answers that are definitely wrong before
confirming why the correct answer is correct.

Answer A says that the argument ignores the possibility that some but
not all types of damage to HC6 lead to AS. This is the correct answer;
the argument mistakenly assumes that there is only one type of damage
to HC6. If this isn't the case, then it could still be true that there
is a correlation between DHC6 and AS, but there are also instances in
which DHC6 is observed with /AS.

Answer B is incorrect because the argument is NOT presuming this. One
of the premises of the argument is that DCH6 and /AS occur together
sometimes, meaning that DCH6 certainly can't be the only cause of AS.

Answer C is incorrect because there is no unrepresentative population
in the stimulus; the argument simply isn't making a generalization
based on a poor sample.

Answer D is incorrect because in order to mistake cause for effect,
the argument would have to be making a causal claim. However, this
isn't what the argument is doing; rather, the argument is making a
lack of a causal claim in its conclusion.

Answer E is incorrect because the argument is not presuming that
correlation implies causation, it's doing the opposite. The argument
is taking a correlation and concluding that there is a LACK OF
causation, so this answer is out.

Does this make sense? Let us know if you have any more questions!

on March 6, 2020

Hi, I have further question on why answer choice E is incorrect. @ravi, you mentioned that E is taking a correlation and implying a lack of causation...but am I wrong to think that, since the author attempted to establish that the correlation was weaker than generally thought, this is proof that the cause and effect relationship isn't there?

I understand why A is correct, it would just help me figure out why E is evidently incorrect to prevent me from making this mistake moving forward. Thank you!!!